Shoot a basketball with a minimum speed at some angle

Click For Summary
SUMMARY

The discussion focuses on determining the optimal angle for shooting a basketball to minimize the initial speed required, represented by the function ##f(\theta)= 1/(a \sin (\theta) \cos (\theta) -b \cos^2 (\theta))##. It is established that when the basket height (b) is zero, the best angle is ##\theta=45\deg##, aligning with the Jabbar sky hook technique. The derivative condition ##df/d\theta = 0## leads to the equation ##\tan (2\theta) = -a/b##, which is solved for specific values of a and b. The discussion also emphasizes the importance of understanding projectile motion and the derivation of the function ##f(\theta)##.

PREREQUISITES
  • Understanding of projectile motion in physics
  • Familiarity with calculus, specifically derivatives
  • Knowledge of basketball shooting techniques, including line drives and parabolic shots
  • Ability to manipulate trigonometric functions in equations
NEXT STEPS
  • Study the derivation of projectile motion equations in physics
  • Learn how to apply calculus to optimize functions
  • Research basketball shooting techniques and their mathematical implications
  • Explore the use of trigonometric identities in solving projectile motion problems
USEFUL FOR

Students studying physics, basketball coaches, mathematicians interested in applied mathematics, and anyone looking to optimize sports performance through mathematical analysis.

McFluffy
Messages
37
Reaction score
1

Homework Statement


You should shoot a basketball at the angle ##\theta## requiring minimum speed. Avoid line drives and rainbows. Shooting from (0, 0) with the basket at (a, b), minimize ##f(\theta)= 1/(a \sin (\theta) \cos (\theta) -b \cos^2 (\theta))##.
(a) If b =0 you are level with the basket. Show that ##\theta=45\deg## is best (Jabbar sky hook).
(b) Reduce ##df\over d\theta## = 0 to ##\tan (2\theta) = -a/b##. Solve when a =b.
(c) Estimate the best angle for a free throw

Homework Equations


I have no idea on how this derived:##f(\theta)= 1/(a \sin (\theta) \cos (\theta) -b \cos^2 (\theta))##, but based on the question, it seems that this is the initial speed ##f##, as a function of angle, ##\theta##? How does one derive this?

The Attempt at a Solution


Coming from a person that only plays basketball when friends invited to play, I don't understand the terminologies that are being used such as, "line drives", "rainbows", and "Jabbar sky hook". So the first thing I did was to google these phrases:
Line drives = a flat shot with no arc, which is a straight line from (0,0) to (a,b)
Rainbows = A type of shot in basketball which has a higher than normal arc as it moves towards the basket, so it's a parabola.
Jabbar sky hook = When you're level at the basket, you throw the ball, this video explains it:

Now that we know the terminologies(I think), we can proceed. So a straight line or a parabolic path is a no. The question first asked me to minimize ##f(\theta)##, setting ##df \over d\theta## = 0 then finding what ##\theta## to plug into ##f(\theta)## is fairly straightforward but I want to know how was ##f(\theta)## derived? I tried sketching what it would look like in the xy plane and find some clues on how to derive ##f(\theta)## but I can't find it.

Other than that,the part where I'm stuck is the formulation part. The calculus part, I can do those. So what now?
 
Physics news on Phys.org
McFluffy said:
How does one derive this?
Eliminate the time from the two projectile equations and get ##y(x)##. Be sure to write the trig functions in the equation in terms of Tan(θ), then solve for v02. You will get ##f(\theta)## to within a constant which is irrelevant because you are minimizing.

On edit: Actually you don't have to express the trig functions in terms of the tangent. That becomes necessary if you want to find the angle of projection given a, b and the initial speed.
 
Last edited:
  • Like
Likes McFluffy
Question: A clock's minute hand has length 4 and its hour hand has length 3. What is the distance between the tips at the moment when it is increasing most rapidly?(Putnam Exam Question) Answer: Making assumption that both the hands moves at constant angular velocities, the answer is ## \sqrt{7} .## But don't you think this assumption is somewhat doubtful and wrong?

Similar threads

  • · Replies 2 ·
Replies
2
Views
2K
  • · Replies 10 ·
Replies
10
Views
4K
  • · Replies 6 ·
Replies
6
Views
2K
  • · Replies 33 ·
2
Replies
33
Views
4K
Replies
8
Views
2K
  • · Replies 1 ·
Replies
1
Views
3K
  • · Replies 3 ·
Replies
3
Views
3K
Replies
2
Views
6K
  • · Replies 11 ·
Replies
11
Views
2K
Replies
1
Views
3K